1. Trang chủ
  2. » Luận Văn - Báo Cáo

lovetoan wordpress com BẤT ĐẲNG THỨC

175 14 0

Đang tải... (xem toàn văn)

Tài liệu hạn chế xem trước, để xem đầy đủ mời bạn chọn Tải xuống

THÔNG TIN TÀI LIỆU

Cấu trúc

  • Các bất đẳng thức cổ điển

    • Bất đẳng thức AM - GM

      • Bất đẳng thức AM - GM

      • Một số ví dụ áp dụng

      • Bài tập

    • Bất đẳng thức Cauchy - Schwarz

      • Bất đẳng thức Cauchy-Schwarz dạng đa thức

      • Bất đẳng thức Cauchy-Schwarz dạng phân thức

      • Các ví dụ minh họa

      • Bài tập

    • Một số bất đẳng thức khác

      • Bất đẳng thức Schur

        • Bất đẳng thức Schur

        • Các trường hợp đặc biệt

        • Bất đẳng thức Schur mở rộng

        • Các ví dụ

      • Bất đẳng thức Holder

        • Bất đẳng thức Holder

        • Trường hợp đặc biệt

        • Ví dụ minh họa

      • Bất đẳng thức Chebyshev

        • Bất đẳng thức Chebyshev

        • Ví dụ minh họa

      • Bài tập

    • Phương pháp quy nạp

      • Lý thuyết

      • Ví dụ minh họa

    • Phương pháp phân tích bình phương SOS

      • Lý thuyết

        • Một số tiêu chuẩn đánh giá

        • Một số biểu diễn cơ sở

      • Các ví dụ

      • Bài tập

    • Phương pháp dồn biến

      • Lý thuyết

      • Ví dụ minh họa

      • Bài tập

    • Các phương pháp chứng minh bất đẳng thức hiện đại

      • Phương pháp p, q, r

        • Lý thuyết

          • Bất đẳng thức Schur

          • Một số biểu diễn đa thức đối xứng ba biến qua p, q, r

          • Một số đánh giá giữa p, q, r

        • Một số ví dụ

        • Bài tập

      • Phương pháp sử dụng tiếp tuyến và cát tuyến

        • Lý thuyết

          • Hàm lồi - Dấu hiệu hàm lồi

          • Bất đẳng thức tiếp tuyến - Bất đẳng thức cát tuyến

        • Các ví dụ minh họa

        • Bài tập

      • Một số chuyên đề

        • Ứng dụng điều kiện có nghiệm của phương trình bậc ba trong chứng minh bất đẳng thức

          • Lý thuyết

            • Mở đầu

            • Một số kết quả

          • Ví dụ minh họa

          • Bài tập

        • Bài toán tìm hằng số tốt nhất trong bất đẳng thức

          • Lý thuyết

          • Ví dụ minh họa

          • Bài tập

        • Các bất đẳng thức cổ điển

          • Bất đẳng thức AM-GM

            • Bất đẳng thức Cauchy-Schwarz

              • Một số bất đẳng thức khác

                • Một số phương pháp chứng minh bất đẳng thức

                  • Phương pháp quy nạp

                    • Phương pháp phân tích bình phương SOS

                      • Phương pháp dồn biến

                      • Phương pháp p, q, r

                      • Phương pháp tiếp tuyến và cát tuyến

                    • Một số chuyên đề

                      • Ứng dụng đều kiện có nghiệm của phương trình bậc ba

                        • Bài toán tìm hằng số tốt nhất

Nội dung

SỞ GIÁO DỤC VÀ ĐÀO TẠO ĐỒNG NAI Trường THPT Chuyên Lương Thế Vinh ——————–o0o——————– NGUYỄN TẤT THU CHUYÊN ĐỀ BẤT ĐẲNG THỨC Biên Hòa 2018 —————– N gu yễ n T ất T hu Mục lục 1 Các bất đẳng thức cổ điển 3 1 Bất đẳng thức AM GM 3 I Bất đẳng thức AM GM 3 II Một số ví dụ áp dụng 5 III Bài tập 13 2 Bất đẳng thức Cauchy Schwarz 19 I Bất đẳng thức Cauchy Schwarz dạng đa thức 19 II Bất đẳng thức Cauchy Schwarz dạng phân thức 19 III Các ví dụ minh họa 19 IV Bài tập 27 3 Một số bất đẳng thức khác 31 I Bất đ.

SỞ GIÁO DỤC VÀ ĐÀO TẠO ĐỒNG NAI Trường THPT Chuyên Lương Thế Vinh ——————–o0o——————– NGUYỄN TẤT THU CHUYÊN ĐỀ BẤT ĐẲNG THỨC Biên Hòa - 2018 —————– Mục lục N gu yễ n Tấ t T hu Các bất đẳng thức cổ điển Bất đẳng thức AM - GM I Bất đẳng thức AM - GM II Một số ví dụ áp dụng III Bài tập Bất đẳng thức Cauchy - Schwarz I Bất đẳng thức Cauchy-Schwarz dạng đa thức II Bất đẳng thức Cauchy-Schwarz dạng phân thức III Các ví dụ minh họa IV Bài tập Một số bất đẳng thức khác I Bất đẳng thức Schur Bất đẳng thức Schur Các trường hợp đặc biệt Bất đẳng thức Schur mở rộng Các ví dụ II Bất đẳng thức Holder Bất đẳng thức Holder Trường hợp đặc biệt Ví dụ minh họa III Bất đẳng thức Chebyshev Bất đẳng thức Chebyshev Ví dụ minh họa IV Bài tập Phương pháp quy nạp I Lý thuyết II Ví dụ minh họa Phương pháp phân tích bình phương SOS I Lý thuyết Một số tiêu chuẩn đánh giá Một số biểu diễn sở II Các ví dụ III Bài tập Phương pháp dồn biến I Lý thuyết II Ví dụ minh họa III Bài tập Các phương pháp chứng minh bất đẳng thức Phương pháp p, q, r I Lý thuyết Bất đẳng thức Schur 3 13 19 19 19 19 27 31 31 31 31 31 31 34 34 34 34 35 35 36 36 38 38 38 42 42 42 42 43 46 48 48 48 51 đại 53 54 54 54 MỤC LỤC 2 Một số biểu diễn đa thức đối xứng ba biến qua p, q, r Một số đánh giá p, q, r II Một số ví dụ III Bài tập Phương pháp sử dụng tiếp tuyến cát tuyến I Lý thuyết Hàm lồi - Dấu hiệu hàm lồi Bất đẳng thức tiếp tuyến - Bất đẳng thức cát tuyến II Các ví dụ minh họa III Bài tập 54 55 55 56 58 58 58 58 59 66 chứng minh bất 68 68 68 68 70 74 75 75 75 82 Tấ t T bất đẳng thức cổ điển 86 Bất đẳng thức AM-GM 86 Bất đẳng thức Cauchy-Schwarz 109 Một số bất đẳng thức khác 124 yễ n Các 68 hu Một số chuyên đề Ứng dụng điều kiện có nghiệm phương trình bậc ba đẳng thức I Lý thuyết Mở đầu Một số kết II Ví dụ minh họa III Bài tập Bài tốn tìm số tốt bất đẳng thức I Lý thuyết II Ví dụ minh họa III Bài tập N gu Một số phương pháp chứng minh bất đẳng thức Phương pháp quy nạp Phương pháp phân tích bình phương SOS Phương pháp dồn biến Phương pháp p, q, r Phương pháp tiếp tuyến cát tuyến 129 129 130 135 148 150 Một số chuyên đề 156 Ứng dụng kiện có nghiệm phương trình bậc ba 156 Bài tốn tìm số tốt 159 Chương Các bất đẳng thức cổ điển §1 Bất đẳng thức AM - GM Bất đẳng thức AM - GM Tấ t I T hu Bất đẳng thức AM − GM bất đẳng thức cổ điển sử dụng nhiều toán chứng a1 + a2 + · · · + an minh bất đẳng thức Ta biết trung bình cộng nsố thực a1 ,a2 , · · · ,an số n √ √ trung bình nhân n số n a1 a2 · · · an (với điều kiện n a1 a2 · · · an tồn tại) Bất đẳng thức AM − GM cho đánh giá trung bình cộng số thực khơng âm trung bình nhân chúng Cụ thể sau: Định lí Cho n số thực không âm a1 , a2 , · · · , an ta có yễ n √ a1 + a2 + · · · + an ≥ n a1 · a2 · · · an n Đẳng thức xảy a1 = a2 = · · · = an N gu Chứng minh Có nhiều cách đề chứng minh bất đẳng thức AM − GM , ta chứng minh bất đẳng thức AM − GM phương pháp quy nạp Trước hết ta chứng minh bất đẳng thức AM − GM cho trường hợp n = Tức là, cần chứng minh a1 + a2 √ ≥ a1 · a2 Bất đẳng thức tương đương với √ √ √ a1 + a2 ≥ a1 a2 ⇔ ( a1 − a2 ) ≥ Bất đẳng thức cuối hiển nhiên Đẳng thức xảy a1 = a2 Tiếp theo ta chứng minh cho trường hợp n = Tức cần chứng minh √ a1 + a2 + a3 + a4 ≥ a1 · a2 · a3 · a4 Áp dụng trường hợp n = ta có a1 + a2 √ ≥ a1 · a2 a3 + a4 √ ≥ a3 · a4 Do a1 + a2 + a3 + a4 = a1 + a2 a3 + a4 √ √ + a1 a2 + a3 a4 √ 2 ≥ ≥ a1 a2 a3 a4 2 BẤT ĐẲNG THỨC AM - GM Nên trường hợp n = chứng minh Tiếp đến ta chứng minh trường hợp n = 3, tức chứng minh √ a1 + a2 + a3 ≥ a1 · a2 · a3 Đặt a4 = a1 + a2 + a3 Áp dụng cho trường hợp n = ta có √ a1 + a2 + a3 + a4 ≥ a1 · a2 · a3 · a4 , hay a1 + a2 + a3 + a1 + a2 + a3 ≥ a1 · a2 · a3 · a1 + a2 + a3 Suy yễ n Tấ t T hu √ a1 + a2 + a3 ≥ a1 · a2 · a3 (đpcm) Để chứng minh cho trường hợp tổng quát ta chứng minh theo hai bước sau: Bước 1: Ta chứng minh bất đẳng thức với n = 2m +) Với m = 1, ta có n = 2nên bất đẳng thức với m = +) Giả sử bất đẳng thức với n = 2m−1 , ta chứng minh bất đẳng thức với n = 2m Tức √ a1 + a2 + · · · + a2m−1 + · · · + an ≥ n a1 a2 · · · an (1) n Đặt a2m−1 +1 + a2m−1 +2 + · · · + a2m a1 + a2 + · · · + a2m−1 ,y= x= m−1 2m−1 Theo giả thiết quy nạp ta có √ √ m−1 x≥ a1 a2 · · · a2m−1 ,y ≥ 2m−1 a2m−1 +1 · · · an hay x+y √ ≥ xy N gu Áp dụng cho trường hợp n = ta có: √ a1 + a2 + · · · + a2m−1 + a2m−1 +1 + · · · + an m ≥ a1 a2 · · · an m Hay (1) chứng minh Bước 2: Ta chứng minh bất đẳng thức với n ≥ với n − Gải sử √ a1 + a2 + · · · + an ≥ n a1 a2 · · · an n Ta chứng minh a1 + a2 + · · · + an−1 √ ≥ n−1 a1 · a2 · · · an−1 n−1 a1 + a2 + · · · + an−1 ÁP dụng bất đẳng thức AM-GM cho n số ta có Thật vậy: Đặt an = n−1 √ a1 + a2 + · · · + an ≥ n a1 a2 · · · an , n hay a1 + a2 + · · · + a1 + a2 + · · · + an−1 n−1 ≥ n n a1 a2 · · · an−1 · a1 + a2 + · · · + an−1 n−1 BẤT ĐẲNG THỨC AM - GM Suy a1 + a2 + · · · + an−1 ≥ n−1 √ n−1 a1 · a2 · · · an−1 (đpcm) Từ hai bước ta có bất đẳng thức AM − GM chứng minh Hệ Cho số thực dương a1 ,a2 , · · · ,an Ta có 1 n2 + + ··· + ≥ a1 a2 an a1 + a2 + · · · + an Đẳng thức xảy a1 = a2 = · · · = an II Một số ví dụ áp dụng Ví dụ 1.1 Cho a,b,c > thỏa a2 + b2 + c2 = Chứng minh hu a5 + b5 + c5 ≥ T Áp dụng bất đẳng thức AM-GM ta có a5 + a5 + + + ≥ 3a2 hay 2a5 + ≥ 3a2 Tấ t Tương tự 2b5 + ≥ 3b2 2c5 + ≥ 3c2 yễ n Cộng ba bất đẳng thức ta có đpcm Nhận xét Ta có toán tổng quát sau: Cho a,b,c > thỏa mãn a + b + c = (hoặc abc = 1) m,n ∈ N,m ≥ n Khi gu am + bm + cm ≥ an + bn + cn (1) N Bất đẳng thức (1) m,n số hữu tỉ dương Và ta tổng qt biến thành k biến Ví dụ 1.2 Cho a,b,c > thỏa a + 4b + 9c = 6.Chứng minh a3 + b + c ≥ Xét x, y, z số thực dương Áp dụng bất đẳng thức AM-GM ta có a3 + 2x3 = a3 + x3 + x3 ≥ 3x2 a, đẳng thức xảy a = x Tương tự ta có: b3 + 2y ≥ 3y b, c3 + 2z ≥ 3y c Đẳng thức xảy b = y, c = z Cộng bất đẳng thức theo vế ta a3 + b3 + c3 ≥ 3(x2 a + y b + z c) − 2(x3 + y + z ) BẤT ĐẲNG THỨC AM - GM Ta chọn x, y, z cho    x=     x + 4y + 9z = a + 4b + 9c =   2 ⇒ y=   x = y = z = t2     z = Do a3 + b3 + c3 ≥ 3t2 (a + 4b + 9c) − 2(x3 + y + z ) = Ví dụ 1.3 Cho a, b, c > thỏa ab + bc + ca = Chứng minh Áp dụng bất đẳng thức AM-GM ta có Cộng ba bất đẳng thức ta có đpcm Tấ t T a3 + b3 + ≥ 3ab b3 + c3 + ≥ 3bc c3 + a3 + ≥ 3ca hu a3 + b3 + c3 ≥ Ví dụ 1.4 Cho số thực dương a, b, c có tổng bình phương Chứng minh gu yễ n ab bc ca + + ≥ c a b Gọi P vế trái bất đẳng thức cần chứng minh, ta có N ab bc ca P = + + c a b 2 2 ab cb c a2 = + + + 2(a2 + b2 + c2 ) c a b 2 2 ab cb c b c a2 = + + + + 2 2 c a a b 2 2 ≥ b + c + a + = a2 b c a2 + c2 b Suy P ≥ Đẳng thức xảy a = b = c = Ví dụ 1.5 Cho a, b, c > a + b + c = abc Chứng minh : a b c + + ≥ b c a Ta có bất đẳng thức cần chứng minh tương đương với: abc a b c + 3+ 3 b c a ≥ a + b + c +6 BẤT ĐẲNG THỨC AM - GM Hay a2 c b a c b + + ≥ a + b + c b2 c a (1) Áp dụng bất đẳng thức Cô si cho ba số ta : 2 a2 c b a a c b a + + c ≥ .c = 3a b2 c2 b2 c Tương tự : b2 a c b c b a2 c + + a ≥ 3b ; + + b ≥ 3c c2 a2 a2 b Cộng ba bất đẳng thức ta có bất đẳng thức (1) Bài tốn chứng minh Đẳng thức xảy ⇔ a = b = c = √ Ví dụ 1.6 Cho a, b, c > Chứng minh : T hu a5 b c + + ≥ a3 + b + c b2 c2 a2 Áp dụng bất đẳng thức Cô si : a5 ab = 2a3 b2 Tấ t a5 + ab2 ≥ b2 Tương tự : yễ n b5 c + bc ≥ 2b ; + ca2 ≥ 2c3 c2 a2 Công bất đẳng thức lại với ta : N gu a5 b c + + ≥ a3 + b3 + c3 + a3 + b3 + c3 − ab2 − bc2 − ca2 b c a2 Nên ta cần chứng minh : a3 + b3 + c3 − ab2 − bc2 − ca2 ≥ ⇔ a3 + b3 + c3 ≥ ab2 + bc2 + ca2 Áp dụng bất đẳng thức Cô si : √ a3 + b3 + b3 ≥ a3 b3 b3 = 3ab2 ⇒ a3 + 2b3 ≥ 3ab2 Tương tự : b3 + 2c3 ≥ 3bc2 ; c3 + 2a3 ≥ 3ca2 Công bất đẳng thức lại với ta có (1) Vậy tốn chứng minh Ví dụ 1.7 Cho số thực dương a,b,c Chứng minh a4 b4 c4 a+b+c + + ≥ b2 (c + a) c2 (a + b) a2 (b + c) (1) BẤT ĐẲNG THỨC AM - GM Áp dụng bất đẳng thức AM-GM ta có b b c+a a4 + + + ≥ 2a b (c + a) 2 hay a4 c+a +b+ ≥ 2a b (c + a) Tương tự, ta có b4 a+b c4 b+c + c + ≥ 2b + a + ≥ 2c c2 (a + b) a2 (b + c) Cộng ba bất đẳng thức theo vế ta có đpcm Ví dụ 1.8 (BĐT Nesbit cho số) Cho a, b, c > Chứng minh hu a b c + + ≥ b+c c+a a+b Hay Ta có c +1 a+b 1 + + a+b b+c c+a ≥ ≥ 9 yễ n (a + b + c) b +1 + c+a Tấ t a +1 + b+c T Bất đẳng thức cần chứng minh tương đương với 1 9 + + ≥ = a+b b+c c+a a+b+b+c+c+a (a + b + c) gu Nên (1) N Ví dụ 1.9 Cho số thực dương a, b, c thỏa a + b + c = Chứng minh Ta có: 1 1 + + + ≥ 30 a2 + b2 + c2 ab bc ca (a + b + c)2 ab + bc + ca ≤ = 3 1 + + ≥ ab bc ca ab + bc + ca 1 + + ≥ = 2 a +b +c ab + bc + ca ab + bc + ca (a + b + c)2 Do + a2 + b2 + c2 ab + bc + ca 1 7 = + + + ≥ + = 30 2 a +b +c ab + bc + ca ab + bc + ca ab + bc + ca VT ≥ Ta có điều phải chứng minh (1) BẤT ĐẲNG THỨC AM - GM Ví dụ 1.10 Cho số thực dương x,y,z thỏa mãn : xy + yz + zx = 3.Chứng minh rằng: + ≥ xyz (x + y)(y + z)(z + x) Ta có: xyz (x + y) (y + z) (z + x) ≤ x (y + z) + y (z + x) + z (x + y) = Suy xyz ≥ (x + y) (y + z) (z + x) Do VT ≥ xyz xyz 1 + ≥ + + ≥1+ = xyz 2xyz 2xyz 2 hu Bài toán chứng minh T Ví dụ 1.11 (IMO 2012) Cho n ≥ số thực dương a2 , a3 , , an thỏa mãn a2 a3 · · · an = Chứng minh k Suy 1 + + ··· + + ak k−1 k−1 k−1 gu (1 + ak ) = yễ n Áp dụng bất đẳng thức AM-GM ta có Tấ t (1 + a2 )2 (1 + a3 )3 · · · (1 + an )n > nn N (1 + a2 )2 (1 + a3 )3 · · · (1 + an )n ≥ k ≥ k k ak (k − 1)k−1 22 33 44 nn · · · · · a1 a2 · · · an = n n 11 22 33 (n − 1)n Ta thấy khơng có đẳng thức xảy Vậy tốn chứng minh Ví dụ 1.12 Cho số thực dương a, b, c có tích Chứng minh 1+ ≥ a+b+c ab + bc + ca Bất đẳng thức cần chứng minh tương đương với ab + bc + ca + 3(ab + bc + ca) ≥ a+b+c Áp dụng bất đẳng thức AM-GM ta có ab + bc + ca + 3(ab + bc + ca) ≥2 a+b+c 3(ab + bc + ca)2 a+b+c (1) BÀI TỐN TÌM HẰNG SỐ TỐT NHẤT Do k − ≤ ⇔ k ≤ Ta chứng minh k = giá trị lớn cần tìm Tức ta cần chứng minh: b c a + + ≥ + 9bc + 4(b − c) + 9ca + 4(c − a) + 9ab + 4(a − b) (1) Áp dụng Bất đẳng thức Cauchy – Schwarz, ta có: (a + b + c)2 (a + b + c) + 27abc + 4a(b − c)2 + 4b(c − a)2 + 4c(a − b)2 = + 3abc + 4ab (a + b) + 4bc (b + c) + 4ca (c + a) VT ≥ Do để chứng minh (1) ta cần chứng minh: ≥ 3abc + 4ab (a + b) + 4bc (b + c) + 4ca (c + a) ⇔ (a + b + c)3 ≥ 3abc + 4ab (a + b) + 4bc (b + c) + 4ca (c + a) ⇔ a3 + b3 + c3 + 3abc ≥ ab (a + b) + bc (b + c) + ca (c + a) gu yễ n Tấ t T hu (điều đúng, bất đẳng thức Schur) Do (1) với a, b, c không âm thỏa a + b + c = với kmax = Vậy k = số cần tìm a Câu 2.3 Thử chọn b = c xét hàm theo biến , ta khơng dẫn đến kết cần tìm Như b ta thử chọn biến 0, biến tiến dần đến Trong bất đẳng thức, ta cho c → 0, a3 + b3 ≥ M ab2 Nhưng lại theo bất đẳng thức AM – GM, ta có b3 b3 a3 + b = a3 + + ≥ √ · ab2 2 3 Như ta thấy M ≤ √ Do ta chứng minh M = √ giá trị lớn cần tìm sau: 3 4 Khơng giảm tính tổng qt, giả sử c = {a,b,c} Đặt a = u + c, b = v + c với u, v ≥ Ta chứng minh: N (u + c)3 + (v + c)3 + c3 − 3c (u + c) (v + c) ≥ M (u + c) (v + c)2 + (v + c) c2 + c(u + c)2 − 3c (u + c) (v + c) ⇔ (3 − M ) u2 − uv + v + u3 + v − M uv ≥ Bất đẳng thức cuối ln vì, − M > 0, u2 − uv + v ≥ uv ≥ u3 + v = u3 + v3 v3 + ≥ √ uv = M uv 2 Từ ta hồn tất phần chứng minh Vậy giá trị lớn cần tìm là: M = √ Câu 2.4 Vì tính bất đẳng thức nên ta chuẩn hóa xyz = Từ ta biến đổi: a b c x = , y = , z = Khi bất đẳng thức trở thành: b c a a2 b2 c2 a b c + + + 3k ≥ (k + 1) + + bc ca ab b c a ⇔ a3 + b3 + c3 + 3k · abc ≥ (k + 1) ab2 + bc2 + ca2 ⇔ a3 + b3 + c3 − 3abc ≥ (k + 1) ab2 + bc2 + ca2 − 3abc 160 BÀI TỐN TÌM HẰNG SỐ TỐT NHẤT Đây bất đẳng thức 3 ⇔ k = −1 + √ Vậy k + = √ 3 4 Câu 2.5 Ta chọn b = c giả thiết trở thành: a + 2b = b2 + 2ab ⇒ a = Vì a > nên 2b − b2 2b − 2b − b2 > ⇔ < b ≤ 2b − Bất đẳng thức trở thành: (a + 2b) + 2b a + b  ≥ k (a + 2b + 1)  1  2b − b2 ≥k + 2b  +   2b − b 2b − 2b +b 2b − 3b − b 2k ⇔ ≥ 3b + 5b + b − 2b − b2 + 2b + 2b − T hu ⇒ yễ n Tấ t 3b2 − b ;2 Xét hàm số f (b) = 3b + 5b2 + b − Ta có − (b2 − 1) (3b − 1)2 f (b) = ⇔ = ⇔ b = (3b3 + 5b2 + b − 1)2 Bằng cách lập bảng biến thiên ta tìm minf (b) = b = Từ tìm k ≤  ;2 gu Ta chứng minh k = giá trị lớn cần tìm Thực ta cần chứng minh: 1 + + ≥a+b+c+1 a+b b+c c+a 1 + + ≥a+b+c+1 ⇔ (ab + bc + ca) a+b b+c c+a ab bc ca + + ≥ ⇔ a+b b+c c+a N (a + b + c) Nhận thấy dấu “=” đạt biến hai biến nên đánh giá bất đẳng thức thơng thường có dấu “=” tâm không dẫn đến kết Để đảm bảo dấu “=” ta loại bỏ dấu “=” biên cách sử dụng abc ≥ sau: Theo Cauchy - Schwarz: ab bc ca (ab + bc + ca)2 + + ≥ a+b b+c c+a ab (a + b) + bc (b + c) + ca (c + a) (ab + bc + ca)2 = (a + b + c) (ab + bc + ca) − 3abc (ab + bc + ca)2 ≥ = (a + b + c) (ab + bc + ca) Chứng minh hoàn thành Ta kết luận số k lớn cần tìm k = 161 BÀI TỐN TÌM HẰNG SỐ TỐT NHẤT Câu 2.6 Cho b = c, ta có bất đẳng thức trở thành: + a b (a + 2b) −9≥k 1− 2ab + b2 a2 + 2b2 (a − b)2 2(a − b)2 ≥k· ab a + 2b2 k a2 + 2b2 ≥ ⇔ ab ⇔ Theo bất đẳng thức AM – GM, ta có: √ a2 + 2b2 ≥ 2 ab √ Suy k ≤ Ta chứng minh giá trị k lớn cần tìm Sử dụng bất đẳng thức Schur – SOS sau: √ 1 ab + bc + ca + + −9≥4 1− a b c a + b2 + c √ a2 + b2 + c2 − ab − bc − ca (a + b + c) (ab + bc + ca) − 9abc ≥4 2· ⇔ abc a2 + b + c √ (a − b)2 + (a − c) (b − c) 2c(a − b)2 + (a + b) (a − c) (b − c) ⇔ ≥4 2· abc a2 + b + c ⇔ M · (a − b)2 + N · (a − c) (b − c) ≥ √ √ 2c a+b Trong đó: M = − ,N= − abc a2 + b2 + c2 abc a + b2 + c Giả sử c = max {a,b,c} việc hoàn tất N ≥ 0, hay √ (a + b) a2 + b2 + c2 ≥ 2abc yễ n Tấ t T hu (a + b + c) Và mạnh nữa: gu Do tính nên ta chuẩn hóa a + b + c = Ta chứng minh kết mạnh sau: √ √ a2 + b2 + c2 ≥ c (a + b) ⇔ − 2c (a + b) ≥ c (a + b) + 2ab √ N (a + b)2 2 √ t ⇔ − · t (1 − t) ≥ · t (1 − t) + , với t = a + b √ √ √ −2 + t 3−2 2 ⇔ t − 2+ t+1≥0⇔ −1 2 − · c (a + b) ≥ · c (a + b) + ≥ Điều Như chứng minh hoàn √ tất Vậy số k lớn thỏa đề k = Câu 2.7 Ta chọn a = b = t, a = b = t, c = − 2t Khi đó, ta có: k 2t4 + (3 − 2t)4 − ≥ 2t3 + (3 − 2t)3 + 3t2 (3 − 2t) − −12t3 + 45t2 − 54t + 21 ⇔k≥ 18t4 − 96t3 + 216t2 − 216t + 78 3(t − 1)2 (7 − 4t) = (t − 1)2 (18t2 − 60t + 78) − 4t = = f (t) 2 (3t − 10t + 13) 162 BÀI TỐN TÌM HẰNG SỐ TỐT NHẤT Ta có: 12 (2t2 − 7t + 3) 3; = ⇔ t ∈ 2 4(3t − 10t + 13) Khoảng giá trị t (0; 3), từ ta thấy minf (t) = t = (0;3) Vậy trở lại toán, ta cho a = b = , c = ta k ≥ 27 , ta chứng minh giá trị nhỏ 2 k cách chứng minh: f (t) = a + b4 + c4 − ≥ a3 + b3 + c3 + 3abc − Đặt f (a,b,c) = V T − V P Ta chứng minh: f (a,b,c) ≥ f (a + b)4 (a + b)3 (a + b)2 a + b4 − − a3 + b − − 3c ab − 4 hu ⇔ a+b a+b , ,c 2 Tấ t T (a − b)4 + 6(a − b)2 (a + b)2 (a + b) (a − b)2 (a − b)2 ⇔ − + 3c · ≥0 28 4 7a2 + 7b2 + 10ab 3c ⇔ − (a + b) + ≥0 28 4 ⇔ a2 + b2 + 10ab − 21 (a + b) + 21c ≥ ⇔ a2 + b2 + 10ab − 21 (a + b) + 21 (3 − a − b) ≥ ⇔ (a + b)2 + a2 + b2 − 42 (a + b) + 63 ≥ yễ n Do (a2 + b2 ) ≥ (a + b)2 nên ta chứng minh: 6(a + b)2 − 42(a + b) + 63 ≥ gu Và điều ta giả sử c = max {a,b,c} a + b ≤ Vậy ta cần ra: a+b a+b , ,c 2 N f ⇔ ≥ ⇔ f (t; t; − t) ≥ 0,t = a+b 2 2t4 + (3 − 2t)4 − ≥ 2t3 + (3 − 2t)3 + 3t2 (3 − 2t) − Bất đẳng thức theo lập luận chặn t phía Vậy k = giá trị nhỏ cần tìm Câu 2.8 Cho b = c giả thiết cho viết thành: a + 2b = b2 + 2ab ⇒ a = b2 − 2b ≥ ⇒ ≤ b ≤ − 2b Lưu ý tìm số thực k lớn nên xét k > Thay vào bất đẳng thức, ta được: b2 − 2b b2 (b2 − 2b) +k ≥k+3 − 2b − 2b ⇔ 2b (1 − 2b) + b2 − 2b + kb2 b2 − 2b ≥ k (1 − 2b) + (1 − 2b) ⇔ (b − 1)2 ≥ k b2 − (b − 1)2 ⇔ ≥ b2 − k 2b + 163 ≥0 BÀI TỐN TÌM HẰNG SỐ TỐT NHẤT Mà b2 − ≤ 22 − = suy k ≤ Ta chứng minh k = giá trị lớn cần tìm Với giả thiết a + b + c = ab + bc + ca, ta cần chứng tỏ ab + bc + ca + abc ≥ Ta phản chứng ab + bc + ca + abc = chứng minh a + b + c ≥ ab + bc + ca Đây kết quen thuộc kì thi VMO 1996 Câu 2.9 Trong bất đẳng thức ban đầu, ta cho b = c, ta được: hu a3 + 2b3 b2 + 2ab k +k· + ≥ 2b (a + b) (a + 2b) a3 + 2b3 1 b2 + 2ab ⇔ − ≥k· − b(a + b)2 (a + 2b)2 (a − b)2 (a − b)2 (4a + 5b) ≥ k · ⇔ 8b(a + b)2 3(a + 2b)2 8k (a + 2b)2 (4a + 5b) ⇔ ≤ b(a + b)2 Do tính nên ta chọn b = Khi đó, ta có: T 8k (a + 2)2 (4a + 5) ≤ (a + 1)2 Tấ t (a + 2)2 (4a + 5) Ta xét hàm f (a) = , a > Ta có: (a + 1)2 N gu yễ n √ −1 + (a + 2) (2a2 + 2a − 1) =0⇔a= f (a) = (do a > 0) (a + 1)3 √ √ √ 9+6 −1 + Lập bảng biến thiên ta f (a) ≥ f = + Suy ra: k ≤ √ 9+6 Ta chứng minh giá trị k lớn cần tìm Ta sử dụng kĩ thuật Schur – SOS phần chứng minh Bất đẳng thức cần chứng minh viết dạng: (a3 + b3 + c3 − 3abc) − ((a + b) (b + c) (c + a) − 8abc) a2 + b2 + c2 − ab − bc − ca ≥k· (a + b) (b + c) (c + a) 3(a + b + c)2 Ta sử dụng khai triển: a3 + b3 + c3 − 3abc = (a + b + c) (a − b)2 + (a − c) (b − c) (a + b) (b + c) (c + a) − 8abc = 2c (a − b)2 + (a + b) (a − c) (b − c) a2 + b2 + c2 − ab − bc − ca = (a − b)2 + (a − c) (b − c) Từ ta nhóm điều cần chứng minh thành: M (a − b)2 + N (a − c) (b − c) ≥ 0, đó: 8a + 8b + 2c k − (a + b) (b + c) (c + a) 3(a + b + c)2 5a + 5b + 8c k N= − (a + b) (b + c) (c + a) 3(a + b + c)2 M= 164 BÀI TỐN TÌM HẰNG SỐ TỐT NHẤT Bây giờ, khơng giảm tổng quát ta giả sử c = {a,b,c} nên a + b ≥ 2c Từ 8a + 8b + 2c ≥ 5a + 5b + 8c Từ ta thấy M ≥ N Hơn có (a − c) (b − c) ≥ Hơn có (a − c) (b − c) ≥ Như bất đẳng thức chứng minh hoàn tất ta N ≥ Tức là: (5a + 5b + 8c) (a + b + c)2 ≥ 8k (a + b) (b + c) (c + a) Đổi biến (a + b,b + c,c + a) → (X,Y,Z) Khi cần chứng minh: (X + 4Y + 4Z) (X + Y + Z)2 ≥ 32k · XY Z Ta chứng minh kết mạnh là: √ (X + 4Y + 4Z) (X + Y + Z)2 ≥ 8k · X (Y + Z)2 = 27 + 18 X (Y + Z)2 Vì tính nên ta chuẩn hóa X + Y + Z = Từ ta cần chứng minh: √ (4 − 3X) ≥ 27 + 18 X (1 − X)2 ⇔ g (X) = hu √ − 3X 3, ≥ + X(1 − X)2 Câu 2.10 Cho b = c ta được: b +k b+a yễ n a +k 2b Tấ t T < X < √ Khảo sát hàm g khoảng (0; 1) ta g (X) ≥ + Như ta hoàn tất chứng minh √ 9+6 Vậy k lớn Cho a → ta được: ≥ k+ √ −1 − k ≤ 4√ 8k (k + 1)2 ≥ (2k + 1)3 ⇔ 4k + 2k ≥ ⇔   −1 + k≥ N gu  Ta chứng minh tồn giá trị k cần tìm Đặt x = 2a 2b 2c ,y= ,z= b+c c+a a+b xy + yz + zx + xyz = Ta cần chứng minh: (x + 2k) (y + 2k) (z + 2k) ≥ (2k + 1)3 ⇔ xyz + 2k (xy + yz + zx) + 4k (x + y + z) + 8k ≥ (2k + 1)3 Theo kết VMO 1996, ta được: x + y + z ≥ xy + yz + zx Từ đó: xyz + 2k (xy + yz + zx) + 4k (x + y + z) + 8k ≥ xyz + 2k (xy + yz + zx) + 4k (xy + yz + zx) + 8k = (xy + yz + zx + xyz) + 4k + 2k − (xy + yz + zx) + 8k = 8k + + 4k + 2k − (xy + yz + zx) 165 BÀI TỐN TÌM HẰNG SỐ TỐT NHẤT Hơn ta có: xy + yz + zx ≥ 3 (xy + yz + zx)3 √ 3 (xyz) ⇒ xyz ≤ Từ đó: = xy + yz + zx + xyz ≤ xy + yz + zx + với t = (xy + yz + zx)3 t3 √ = t2 + √ , 3 3 √ √ xy + yz + zx Suy ra: t ≥ 3⇒ xy + yz + zx ≥ Suy ra: xyz + 2k (xy + yz + zx) + 4k (x + y + z) + 8k ≥ 8k + + 4k + 2k − (xy + yz + zx) ≥ 8k + + 4k + 2k − = (2k + 1)3 Bài toán hoàn tất T hu Câu 2.11 Trước hết ta tìm điều kiện cần cho k Thay a = b = x, c = với x > 0, x = 1, bất đẳng thức cho viết lại thành x k k ≥3+ 2x + + x x2 − ⇔ x2 + − ≥ k x 2x + x2 + + x Tấ t x2 + gu yễ n (x − 1)2 (x + 2) k(x − 1)2 (2x + 1) ≥ x (2x3 + x2 + 1) k (x + 2) (2x3 + x2 + 1) ⇔ ≤ x (2x + 1) k ⇔ ≤ x2 + 2x + − , ∀x > 0, x = x 2x + ⇔ N √ 3+1 tính giá trị biểu thức Bây giờ, ta cho x = f (x) = x2 + 2x + − x 2x + bên vế phải Để ý x nghiệm phương trình, dó ta có x2 = = − 4x Suy 2x + 4x + 1 , = 8x − 4, x 4x + + 2x + (8x − 4) − (4 − 4x) √ √ 45 95 45 + 95 45 − 50 = · − = = x− 8 f (x) = Với kết này, ta thu √ √ 45 − 50 k≤ = 45 × − 25 ≈ 45 × 0.866 − 25 ≈ 13.97 2 166 BÀI TỐN TÌM HẰNG SỐ TỐT NHẤT Mặt khác, k số ngun nên từ có k ≤ 13 Tiếp theo, ta chứng minh k = 13 thỏa mãn yêu cầu toán, tức 1 13 25 + + + ≥ a b c a+b+c+1 Đặt f (a,b,c) = 1 13 + + + a b c a+b+c+1 Khơng tính tổng qt, giả sử a = max{a,b,c}, ta có √ √ f (a,b,c) − f a, bc, bc = 1 1 √ + −√ + 13 − b c a + b + c + a + bc + bc   √ √ 13  = b− c  − bc (a + b + c + 1) a + 2√bc + Do a = max{a; b; c} giả thiết abc = nên ta có bc ≤ 1, suy ≥ Mặt khác, sử dụng bất bc 13 ≤ √ f abc + √ 3 abc + = 13 < 16 Tấ t Từ ta đưa toán chứng minh 13 √ T (a + b + c + 1) a + bc + hu đẳng thức AM - GM, ta lại có ,x,x x2 ≥ 25 N gu yễ n √ với x = bc, < x ≤ Nếu x = bất đẳng thức trở thành đẳng thức Trong trường hợp < x < cách sử dụng lại biến đổi thực trình tìm điều kiện cho k, ta thấy bất đẳng thức tương đương với 13 (x + 2) (2x3 + x2 + 1) ≥ x (2x + 1) ⇔ (x + 2) 2x + x + ≥ 13x (2x + 1) ⇔ 2x4 + 5x3 + 2x2 + x + ≥ 26x2 + 13x ⇔ 8x4 + 20x3 − 18x2 − 9x + ≥ Ta có 8x4 + 20x3 − 18x2 − 9x + = 8x4 − 8x2 + + 20x3 − 20x2 + 5x + 10x2 − 14x + = 2x2 − + 5x (2x − 1)2 + 5x2 − 7x + > Do (2x2 − 1) ≥ 0,5 · x (2x − 1)2 ≥ 5x2 − 7x + > (tam thức bất hai có hệ số cao dương biệt thức ∆ = −11 < 0) Như vậy, bất đẳng thức cuối hiển nhiên Vậy k = 13 giá trị cần tìm 167 BÀI TỐN TÌM HẰNG SỐ TỐT NHẤT Câu 2.12 Cho a = b = t > 0, c = ta t2 2t6 + t3 + 2 + t ⇔ + 3k ≥ (k + 1) + 3k ≥ (k + 1) · t4 t t4 t 6 3 t + 2t + −t + 2t3 − t +2 3t − t − ≥ − ≥ ⇔ k 3− ⇔ k t t t4 t t4 2t2 + (t3 − 1) ⇔ k (t − 1) (t + 2) ≤ t3 2 (t + t + 1) ⇔ k (t + 2) ≤ t3 (t2 + t + 1) ⇔k≤ t4 + 2t3 Đặt z = Khi t (z + z + 1) 2z + Khi t → +∞ z → Như T k≤ hu ⇔k≤ 1 + t t 1+ t 1+ kmax = f (z) , với f (z) = (z + z + 1) , với z ≥ Ta có 2z + Tấ t [0;+∞) gu yễ n (z + z + 1) (2z + 1)2 − (z + z + 1) (2z + 1)2 (z + z + 1) (2z + 1)2 − = ≥ 0, ∀z ≥ (2z + 1)2 f (z) = N Suy hàm số f đồng biến [0; +∞), với z ≥ f (z) ≥ f (0) = Do k ≤ Từ kmax = 1 + + a b c ⇔ a2 + b2 + c2 + ≥ (ab + bc + ca) (do abc = 1) ⇔ a2 + b2 + c2 + 2abc + ≥ (ab + bc + ca) a2 + b + c + ≥ Đây kết quen thuộc chứng minh nguyên lí Diricle (ta dùng dồn biến để chứng minh) Dấu đẳng thức xảy (a,b,c) = (1; 1; 1); a = b → +∞, c → hoán vị Câu 2.13 Thơng thường dạng tốn này, đẳng thức đạt giá trị tâm (1; 1; 1) cịn đạt hai biến Như vạy để chặn k, ta chọn a = b = t, c = 3−2t Ta có: + − ≥ k 2t2 + (3 − 2t)2 − t − 2t − 4t + t − (3t − 2t2 ) ⇔ ≥ k 6t2 − 12t + 3t − 2t2 6t2 − 12t + ⇔k≤ ⇔k≤ 2 (3t − 2t ) (6t − 12t + 6) t (3 − 2t) 168 BÀI TỐN TÌM HẰNG SỐ TỐT NHẤT Như giá trị c = − 2t lớn cần tìm giá trị lớn hàm số f (t) = ,t∈ t (3 − 2t) 0; Theo bất đẳng thức AM - GM, ta có t (3 − 2t) = Từ kết luận kmax = 1 (2t + − 2t)2 · 2t (3 − 2t) ≤ · = 2 8 chứng tỏ 1 + + −3≥ a + b2 + c − a b c Đặt f (a,b,c) = (1) 1 + + − − (a2 + b2 + c2 − 3) Xét hiệu a b c a+b a+b , ,c 2 1 = + + −3− a + b2 + c − − a b c (a + b)2 − − +3+ + c2 − a+b c = Tấ t T hu f (a,b,c) − f 1 + − a b a+b − a2 + b − (a + b)2 gu yễ n (a + b)2 − 4ab a2 + b2 − 2ab = − · ab (a + b) = (a − b)2 − ab (a + b) N Ta giả sử c = max {a,b,c} Khi a + b ≤ 2, suy ab (a + b) ≤ (a + b)3 ≤ < 4 f (a,b,c) ≥ f a+b a+b , ,c 2 Như Ta cần chứng minh f a+b a+b , ,c 2 ≥ ⇔ f (t,t,3 − 2t) ≥ 0, với t = a+b Tức chứng minh + −3≥ 2t2 + (3 − 2t)2 − t − 2t 24t − 16t2 − ⇔ ≤ ⇔ ≤0 t (3 − 2t) t (3 − 2t) (4t − 3)2 16t2 − 24t + ⇔ ≥0⇔ ≥ (đúng) t (3 − 2t) t (3 − 2t) Vậy (∗) chứng minh Từ kết luận kmax = 169 BÀI TỐN TÌM HẰNG SỐ TỐT NHẤT Câu 2.14 Kí hiệu (1) bất đẳng thức nêu đề Giả sử k số thực cho bất đẳng thức (1) với ba số thực a, b, c, mà abc ≥ Trong (1) thay a = 0, b = c = 2, ta + 8k ≥ Suy k ≥ Ta chứng minh k = giá trị nhỏ cần tìm, tức chứng minh với ba số thực không âm a, b, c, ta ln có abc + (a − b)2 + (b − c)2 + (c − a)2 + ≥ a + b + c hay a2 + b2 + c2 + 2abc + ≥ ab + bc + ca + (a + b + c) (2) Xét số (b − 1) (c − 1), (c − 1) (a − 1), (a − 1) (b − 1), ta có a (b − 1) (c − 1) · b (c − 1) (a − 1) · c (a − 1) (b − 1) = abc (a − 1)2 (b − 1)2 (c − 1) ≥ hu Suy có số số nêu không âm Không tính tổng quát, giả sử a (b − 1) (c − 1) ≥ Khi đó, ta có abc ≥ a (b + c − 1) Do vậy, bất đẳng thức (2) chứng minh, ta chứng minh T a2 + b2 + c2 + 2a (b + c − 1) + ≥ ab + bc + ca + (a + b + c) , hay a2 − (4 − b − c) a + b2 + c2 − bc − (b + c) + ≥ 0, Tấ t (3) với a, b, c, mà abc ≥ Ta xem vế trái (3) tam thức bậc hai theo ẩn a, ta có: yễ n ∆ = (b + c − 4)2 − 4b2 − 4c2 + 4bc + (b + c) − 16 = −3b2 − 3c2 + 6bc = −3 (b − c)2 ≤ 0, gu với b, c nên (3) với a, b, c, mà abc ≥ Vậy k = giá trị cần tìm, theo yêu cầu đề N Câu 2.15 Rõ ràng, giá trị x = y = z = thỏa mãn ràng buộc nêu đề Vì thế, bất đẳng thức đề bài, cho x = y = z = ta k + ≥ Suy k ≥ Tiếp theo, ta chứng minh với k = 1, bất đẳng thức đề bất đẳng thức đúng; tức ta chứng minh x+y+z 2 (x + 1) (y + 1) (z + 1) ≤ + 1, (1) với số thực dương x, y, z mà {xy,yz,zx} ≥ Thật vậy, trước hết, ta chứng minh nhận xét sau: Nhận xét: Với a, b hai số thực dương thỏa mãn ab ≥ 1, ta ln có: a+b a2 + b + ≤ +1 Chứng minh: Vì ab ≥ nên a +1 2 a+b 2 − ≥ ab − ≥ Do b + = (ab − 1) + (a + b) ≤ a+b 170 2 −1 + (a + b) = a+b 2 +1 BÀI TỐN TÌM HẰNG SỐ TỐT NHẤT Nhận xét chứng minh Không tổng quát, giả sử x ≤ y ≤ z Khi đó, từ ràng buộc đề bài, suy x ≥ x+y+z Đặt t = Ta có xt = x2 + xy + xz 1+1+1 x (x + y + z) = ≥ = 3 Do đó, áp dụng nhận xét cho cặp (x,t) cặp (y,z), ta 2 t +1 ≤ x +1 x+t 2 y+z 2 +1 2 z +1 ≤ y +1 , (2) (3) +1 x +1 z +1 y +1 t +1 ≤ x+t 2 +1 y+z 2 +1 (4) x+t y+z √ √ · ≥ xt · yz ≥ Do đó, theo nhận xét, ta có 2 2 +1 Từ (4) (5), suy (x2 + N +1 y2 + gu x2 + Do 2 y+z yễ n x+t Tấ t Nhận thấy 2 T hu Nhân (2) (3), vế theo vế, ta 1) (y + 1) (z ≤ x+y+z+t z2 + t2 + ≤ t2 + 1) (t2 x+y+z + + 1) ≤ 4 +1 (5) + Suy (1) chứng minh thế, giá trị k nhỏ cần tìm theo yêu cầu k = Câu 2.16 Kí hiệu (1) bất đẳng thức nêu đề Giả sử k số thực cho bất đẳng thức (1) với ba số thực a, b, c độ dài ba cạnh tam giác Trong (1) thay b = c > ta a 2b 2ab + b2 + +k· ≤ + k 2b a + b a + 2b2 k ≤ + k Suy k ≥ 2 Ta chứng minh k = giá trị nhỏ cần tìm, tức chứng minh với ba số thực a, b, c độ dài ba cạnh tam giác, ta ln có Cố định b cho a tiến tới 0+ ta + a b c ab + bc + ca + + + ≤ b + c c + a a + b a + b2 + c 2 171 (2) BÀI TỐN TÌM HẰNG SỐ TỐT NHẤT Thật vậy, ta có: (2) ⇔ 1− a b+c + 1− b c+a + 1− c a+b ≥ ab + bc + ca + a2 + b + c b+c−a c+a−b a+b−c (a + b + c)2 ⇔ + + ≥ b+c c+a a+b (a2 + b2 + c2 ) (b + c − a)2 (c + a − b)2 (a + b − c)2 (a + b + c)2 ⇔ + + ≥ (b + c) (b + c − a) (c + a) (c + a − b) (a + b) (a + b − c) (a2 + b2 + c2 ) (3) Do a, b, c độ dài ba cạnh tam giác nên b + c − a > 0, c + a − b > 0, a + b − c > Do tất phân thức nằm vế trái (3) có mẫu thức dương Vì thế, kí hiệu V T biểu thức nằm vế trái (3), theo bất đẳng thức Cauchy – Schwarz dạng Engel, ta có: (a + b + c)2 (a2 + b2 + c2 ) VT ≥ Vì hu (b + c − a) + (c + a − b) + (a + b − c) = a + b + c T (b + c − a) (b + c) + (c + a − b) (c + a) + (a + b − c) (a + b) = a2 + b2 + c2 Tấ t Nên (3) chứng minh (2) chứng minh Vậy k = giá trị cần tìm, theo yêu cầu đề thỏa mãn ràng buộc nêu đề Vì thế, 9k + ≤ , mà k > nên suy bất đẳng thức đề bài, cho x = y = z = ta 3k + 2 k ≥ Tiếp theo, ta chứng minh với k = , bất đẳng thức đề bất đẳng thức đúng; tức ta chứng minh 1 + + ≤ 3, (1) 1 x+y+ y+z+ z+x+ 3 √ √ √ với số thực dương x, y, z mà xy + yz + zx = Thật vậy, đặt √ √ √ T = x + y + z = x + y + z + N gu yễ n Câu 2.17 Rõ ràng, giá trị x = y = z = Áp dụng bất đẳng thức Cauchy-Schwarz cho hai số dương, ta có x+y+ 1 + +z 3 ≥ √ 1 √ √ x· √ + y· √ + √ · z 3 hay x+y+ ≤ 3z + T Tương tự ta có: y+z+ ≤ 3x + ; T z+x+ 172 ≤ 3y + T = T BÀI TỐN TÌM HẰNG SỐ TỐT NHẤT Cộng bất đẳng thức vừa nêu trên, vế với vế, với lưu ý T = x + y + z + 2, ta x+y+ + y+z+ + z+x+ ≤ 3z + + 3x + + 3y + = T (1) chứng minh Vậy k = giá trị cần tìm, theo yêu cầu đề Câu 2.18 Kí hiệu (1) bất đẳng thức cần chứng minh Trong (1) cho a = b = c = 1, suy k ≤ 729 Ta chứng minh (1) với k = 729 Từ giả thiết ta có 3 |abc| ≥ |ab| + |bc| + |ca| ≥ (abc)2 ⇒ abc ≥ Do ≥ a2 + |bc| c + a2 + b b + a2 + c a2 + b + c b2 + |ac| c2 + |ab| hu ≥ 93 (abc)2 ≥ 729 Sử dụng bất đẳng thức Cauchy cho chín số a2 số |bc| Vậy max k = 729 Tấ t T Câu 2.19 Gọi (1) bất đẳng thức cần tìm Trong (1) cho a1 = 1; a2 = · · · = an = − ta n−1 n c ≤ n Ta chứng minh BĐT (1) với c = Tức cần chứng minh yễ n n |ai − aj | ≥ 1≤i

Ngày đăng: 15/04/2022, 19:51

TỪ KHÓA LIÊN QUAN

TÀI LIỆU CÙNG NGƯỜI DÙNG

TÀI LIỆU LIÊN QUAN

w